Question and Answers Forum

All Questions      Topic List

Trigonometry Questions

Previous in All Question      Next in All Question      

Previous in Trigonometry      Next in Trigonometry      

Question Number 18892 by Tinkutara last updated on 01/Aug/17

If the equation sin6x + cos4x = −2 have  a family of nonnegative solutions x_k ′s,  where 0 ≤ x_1  < x_2  < x_3  < .... < x_k  < x_(k+1)   ....., then the value of (1/π)Σ_(k=1) ^(1000) ∣x_(k+1)  − x_k ∣ is

$$\mathrm{If}\:\mathrm{the}\:\mathrm{equation}\:\mathrm{sin6}{x}\:+\:\mathrm{cos4}{x}\:=\:−\mathrm{2}\:\mathrm{have} \\ $$ $$\mathrm{a}\:\mathrm{family}\:\mathrm{of}\:\mathrm{nonnegative}\:\mathrm{solutions}\:{x}_{{k}} '\mathrm{s}, \\ $$ $$\mathrm{where}\:\mathrm{0}\:\leqslant\:{x}_{\mathrm{1}} \:<\:{x}_{\mathrm{2}} \:<\:{x}_{\mathrm{3}} \:<\:....\:<\:{x}_{{k}} \:<\:{x}_{{k}+\mathrm{1}} \\ $$ $$.....,\:\mathrm{then}\:\mathrm{the}\:\mathrm{value}\:\mathrm{of}\:\frac{\mathrm{1}}{\pi}\underset{{k}=\mathrm{1}} {\overset{\mathrm{1000}} {\sum}}\mid{x}_{{k}+\mathrm{1}} \:−\:{x}_{{k}} \mid\:\mathrm{is} \\ $$

Answered by ajfour last updated on 01/Aug/17

sin 6x+cos 4x=−2  ⇒ sin 6x=−1  and  cos 4x=−1  ⇒ 6x=2nπ+((3π)/2)     ; n≥0   (here)   or  x=((nπ)/3)+(π/4)   ...(i)  And   4x=2mπ+π    ; m≥0         or  x=((mπ)/2)+(π/4)    ...(ii)  since both (i) and (ii) together  must be true,     x−(π/4)= ((nπ)/3)=((mπ)/2)  or   x−(π/4)=kπ    x_(k+1) −x_k =π  ⇒  (1/π)Σ_(k=1) ^(1000) ∣x_(k+1) −x_k ∣=((1000π)/π)=1000 .

$$\mathrm{sin}\:\mathrm{6x}+\mathrm{cos}\:\mathrm{4x}=−\mathrm{2} \\ $$ $$\Rightarrow\:\mathrm{sin}\:\mathrm{6x}=−\mathrm{1}\:\:\mathrm{and}\:\:\mathrm{cos}\:\mathrm{4x}=−\mathrm{1} \\ $$ $$\Rightarrow\:\mathrm{6x}=\mathrm{2n}\pi+\frac{\mathrm{3}\pi}{\mathrm{2}}\:\:\:\:\:;\:\mathrm{n}\geqslant\mathrm{0}\:\:\:\left(\mathrm{here}\right) \\ $$ $$\:\mathrm{or}\:\:\mathrm{x}=\frac{\mathrm{n}\pi}{\mathrm{3}}+\frac{\pi}{\mathrm{4}}\:\:\:...\left(\mathrm{i}\right) \\ $$ $$\mathrm{And}\:\:\:\mathrm{4x}=\mathrm{2m}\pi+\pi\:\:\:\:;\:\mathrm{m}\geqslant\mathrm{0} \\ $$ $$\:\:\:\:\:\:\:\mathrm{or}\:\:\mathrm{x}=\frac{\mathrm{m}\pi}{\mathrm{2}}+\frac{\pi}{\mathrm{4}}\:\:\:\:...\left(\mathrm{ii}\right) \\ $$ $$\mathrm{since}\:\mathrm{both}\:\left(\mathrm{i}\right)\:\mathrm{and}\:\left(\mathrm{ii}\right)\:\mathrm{together} \\ $$ $$\mathrm{must}\:\mathrm{be}\:\mathrm{true}, \\ $$ $$\:\:\:\mathrm{x}−\frac{\pi}{\mathrm{4}}=\:\frac{\mathrm{n}\pi}{\mathrm{3}}=\frac{\mathrm{m}\pi}{\mathrm{2}} \\ $$ $$\mathrm{or}\:\:\:\mathrm{x}−\frac{\pi}{\mathrm{4}}=\mathrm{k}\pi\:\: \\ $$ $$\mathrm{x}_{\mathrm{k}+\mathrm{1}} −\mathrm{x}_{\mathrm{k}} =\pi \\ $$ $$\Rightarrow\:\:\frac{\mathrm{1}}{\pi}\underset{\mathrm{k}=\mathrm{1}} {\overset{\mathrm{1000}} {\sum}}\mid\mathrm{x}_{\mathrm{k}+\mathrm{1}} −\mathrm{x}_{\mathrm{k}} \mid=\frac{\mathrm{1000}\pi}{\pi}=\mathrm{1000}\:. \\ $$

Commented byajfour last updated on 01/Aug/17

is it correct?

$$\mathrm{is}\:\mathrm{it}\:\mathrm{correct}? \\ $$

Commented byTinkutara last updated on 01/Aug/17

Thank you very much ajfour Sir!

$$\mathrm{Thank}\:\mathrm{you}\:\mathrm{very}\:\mathrm{much}\:\mathrm{ajfour}\:\mathrm{Sir}! \\ $$

Terms of Service

Privacy Policy

Contact: info@tinkutara.com